A principal of $1500 is invested at 8.5% interest, compounded annually.

How much will the investment be worth after 14 years?

Use the calculator provided and round your answer to the nearest dollar.

Answers

Answer 1

The investment will be worth approximately $4468 after 14 years.

What is annual interest rate?

Annual interest rate is the rate at which an investment or loan grows in value over the course of a year, expressed as a percentage. It represents the cost of borrowing or the return on investment over one year.

According to question:

The following equation can be used to determine the future value of an investment using compound interest:

FV = P * (r/n + 1)(n*t)

Where:

FV is the investment's future value.

The principal is P. (initial investment)

The annual interest rate is represented by r,

The number of times the interest is compounded annually by n (expressed as a decimal).

t is the time (in years)

In this case:

P = $1500

r = 8.5% = 0.085

n = 1 (compounded annually)

t = 14

With these values entered into the formula, we obtain:

FV = 1500 * [tex](1 + 0.085/1)^(1*14)[/tex]

     = $4467.56

Therefore, the investment will be worth approximately $4468 after 14 years.

To know more about annual interest rate visit:

https://brainly.com/question/30573341

#SPJ1


Related Questions

What is 47/ 30 in mixed numbers I'm giving 10 points must hurry

Answers

Answer:

1  17/30.

Step-by-step explanation:

you can find this out by seeing how many 30s go into 47 which is 1.

Then subtract to find how much is leftover still over 30.

47 - 30 = 17

So 1 and 17/30

14. The local credit union is offering a special student checking account. The monthly cost of the account is $15. The first 10 checks are free, and each additional check costs $0.75. You search
the Internet and find a bank that offers a student checking account with no monthly charge. The first 10 checks are free, but each additional check costs $2.50.

a. Assume that you will be writing more than 10 checks a month. Let n represent the number of checks written in a month. Write a function rule for the cost c of each account in terms of n.

b. Write an inequality to determine what number of checks in the bank account would be more expensive than the credit union account.

c. Solve the inequality in part b.

Answers

Answer: a.     c(n) = 15 + 0.75(n - 10)

b.   15 + 0.75(n - 10) = 2.50(n - 10)=

Simplifying and solving for n, we get:

n = 50

c.  n > 50

Step-by-step explanation:

a. The cost c of the credit union account in terms of the number of checks written n can be expressed as:

c(n) = 15 + 0.75(n - 10)

The first term, 15, represents the monthly cost of the account, and the second term represents the additional cost per check beyond the first 10 free checks.

The cost c of the bank account in terms of the number of checks written n can be expressed as:

c(n) = 2.50(n - 10)

The first term, 0, represents the monthly cost of the account, and the second term represents the additional cost per check beyond the first 10 free checks.

b. We want to find the number of checks for which the bank account is more expensive than the credit union account. Let x be the number of checks that makes the cost of the two accounts equal. Then we have:

15 + 0.75(n - 10) = 2.50(n - 10)

Simplifying and solving for n, we get:

n = 50

So if the number of checks written in a month is greater than 50, the bank account will be more expensive than the credit union account.

c. The solution to the inequality is:

n > 50

This means that the number of checks written in a month must be greater than 50 for the bank account to be more expensive than the credit union account.

please help me fill these boxes

Answers

The measurements for area of Jacobs yard are;

Part A = 6m x 3m = 18m

Part B = 4.5m x 3m = 13 m

Part C = 1/2 x 3m x 3m = 4.5 m²

Total area = 18m² + 13.5m² + 4.5m² = 36m²

How do you identify sections that would help in calculating area?

To identify sections that would help in calculating area, you need to look for shapes or figures that can be divided into simpler geometric shapes, such as squares, rectangles, triangles, and circles.

Once you have identified the simpler shapes, you can use their formulas to calculate their areas and then add them together to find the total area of the larger shape or figure.

For example, a rectangle can be divided into two triangles or two smaller rectangles, and a circle can be divided into a sector or a ring. Breaking down a larger shape into smaller, simpler shapes can make it easier to calculate their areas accurately.

Jacob is putting tiles on the section of his yard labeled A, B, C. What is the area of the parts that need tiles?

Part A = .............. x ........... = ...........m

Part B = + .............. x ............. = ............ m

Part C = 1/2 x ................. x ............ = ............... m²

Total area = ................... + ..................... + .................. = ..............m

Find more exercises on calculating area;

https://brainly.com/question/30656333

#SPJ1

Evaluate (2-5i)(p+q)(i) when p=2 and q=5i

Answers

Answer:

29i

Step-by-step explanation:

While multiplying complex numbers we should remember that i^2=−1.

As p=2 and q=5i,

(2−5i)(p+q)i

= (2−5i)(2+5i)i

= (2×2+2×5i−5i×2−5i×5i)×i

= (4+10i−10i−25×i2)×i

= (4+10i−10i−25×(−1))×i

= (4+25)×i

=29i

A partial table of nutrients and Daily Values (DVS)
based on a 2000-calorie diet is provided. The Sodium row and the Vitamin D row are completed, and each % of the DV is calculated.
Compare each amount with the amount on the given nutrition label. Now use the amount of
saturated fat on the nutrition label to calculate its
% of DV, X. Use the saturated fat amount on the nutrition label
to calculate the %DV for saturated fat.

Answers

Note that the %DV for saturated fat in this 2 tbsp serving size is approximately 18%.

What is the explanation for the above response?

To calculate the %DV for saturated fat, we need to first calculate how many grams of saturated fat are in the 2 tablespoon (tbsp) serving size.

From the label, we see that the serving size contains 3.5g of saturated fat.

To calculate the %DV for saturated fat, we use the equation:

%DV = (amount of nutrient per serving / DV) x 100%

Plugging in the values for saturated fat, we get:

%DV = (3.5g / 19g) x 100%

%DV = 0.1842 x 100%

%DV ≈ 18%

Therefore, the %DV for saturated fat in this 2 tbsp serving size is approximately 18%.

Learn more about %DV at:

https://brainly.com/question/14046971

#SPJ1

how did slugger mcfist get a black eye

Answers

He was hit by a guided muscle

Please help me i really need this done

Answers

Answer:

Step-by-step explanation:

4-10(9m-7)

4-90m+70

74-90m

56x+24/8

8(7x + 3)/8

7x + 3

24r + 16

8(3r + 2)

is 2m-3 same as 4(1/2m-3)

2m-3 = 2m-12

: no

-2(10-15x) same as 14x-20+16x

-20+30x = -20+30x

: yes

Use the box plot showing the ages of those who watch the television show 'The Code" to answer the question that follows.
Which value is the best approximation for the range in ages for the middle 50% of viewers?
A) 10
B) 15
C) 20
D) 45

Answers

The range in ages for the middle 50% of viewers is the interquartile range (IQR), which is the height of the box in the box plot. The best approximation is C) 20.

What is interqurtile range?

The interquartile range (IQR) is a measure of statistical dispersion that represents the difference between the 75th percentile (Q3) and the 25th percentile (Q1) of a dataset. It is a useful measure of spread because it is not influenced by outliers.

What is Range?

Range is a statistical measure that represents the difference between the highest and lowest values in a set of data. It provides a simple indication of the spread or variability of the data.

According to the given information:

A box plot is a graphical representation of the distribution of a dataset. The box in the plot represents the middle 50% of the data, with the lower end of the box representing the 25th percentile (Q1) and the upper end of the box representing the 75th percentile (Q3). The distance between Q1 and Q3, which is represented by the height of the box, is called the interquartile range (IQR).

To answer the question, we need to find the best approximation for the range in ages for the middle 50% of viewers. From the box plot, we can see that the height of the box is approximately 20 units, which is the IQR. Therefore, the best approximation for the range in ages for the middle 50% of viewers is option C) 20. This means that 50% of viewers are between Q1-10 to Q3+10, where Q1 is the 25th percentile and Q3 is the 75th percentile.

To know more about interquartile range and range visit:

https://brainly.com/question/29204101

#SPJ1

Use the graph to answer the questions
WILL MARK BRAINLIEST!!

Answers

The diagram of the Gateway Arch on the coordinate plane, analyzed using quadratic equations indicates;

1. The vertex point is (50, 630)

2. The solution point are; (20, 0), and (80, 0)

3. Vertex form; f(x) = -0.7·(x - 50)² + 630

4. Factored form; f(x) = -0.7·(x - 20)·(x - 80)

What is a quadratic equation?

A quadratic equation is an equation of the form f(x) = a·x² + b·x + c

1. The vertex obtained from the graphical diagram of the Gateway Arch indicates that the point corresponding to the vertex point is; (50, 9 × 70 = 630)

The vertex point is; (50, 630)

2. The solution are the points the curve of the Gateway intersects the x-axis, which are points where the y-axis values are zero, therefore;

The solutions are; (20, 0), and (80, 0)

3. The vertex form of a quadratic equation is; f(x) = a·(x - h)² + k

Where;

(h, k) = The coordinates of the vertex

Therefore;

(h, k) = (50, 630)

f(20) = 0 = a·(20 - 50)² + 630

a·(20 - 50)² = -630

a = -630/((20 - 50)²) = -630/900 = -7/10

a = -7/10 = -0.7

The vertex form quadratic equation is therefore; f(x) = -0.7·(x - 50)² + 630

4. The factored form of a quadratic equation is; f(x) = a·(x - r₁)·(x - r₂)

r₁ = 20, and r₂ = 80, a obtained from the vertex is; a = -0.7

The factored form is therefore; f(x) = -0.7·(x - 20)·(x - 80)

Learn more on the factored form of a quadratic equation here: https://brainly.com/question/25094938

#SPJ1

Work out sheet below please

Answers

Answer:

-4 + 8 = 4

-2 + 6 = 4

-1 + 5 = 4

So the three pairs are -4 and 8; -2 and 6; and -1 and 5.

LOOK AT THE PHOTO PLS

Answers

The next entry on the long division would be 0.054, and 0.0054

How to perform long division

Long division is a method of dividing two numbers using a step-by-step process. Here's how to perform long division:

Step 1: Write the dividend (the number being divided) and the divisor (the number you're dividing by) in the long division format, with the dividend inside the division symbol and the divisor outside.

Read more on long division here:https://brainly.com/question/25289437

#SPJ1

A hiker on the Appalachian Trail planned to increase the distance covered by 10% each day. After 7 days, the total distance traveled is 56.923 miles.

Part A: How many miles did the hiker travel on the first day? Round your answer to the nearest mile and show all necessary math work. (4 points)

Part B: What is the equation for Sn? Show all necessary math work. (3 points)

Part C: If this pattern continues, what is the total number of miles the hiker will travel in 14 days? Round your answer to the hundredths place and show all necessary math work. (3 points)

Answers

The hiker traveled approximately 4 miles on the first day.The equation for Sn would be Sn = x(1 - r^n)/(1 - r).The hiker will travel approximately 167.63 miles in 14 days.

Geometric series

Let x be the distance traveled on the first day. Then, the distance traveled on the second day is 1.1x, on the third day is 1.1(1.1x) = 1.21x, and so on. After 7 days, the total distance traveled is:

x + 1.1x + 1.21x + ... + (1.1)^6 x = 56.923

Using the formula for the sum of a geometric series, we have:

x(1 - (1.1)^7)/(1 - 1.1) = 56.923

x(1 - 1.1^7)/(-0.1) = 56.923

x = 56.923(-0.1)/(1 - 1.1^7) ≈ 4 miles

Therefore, the hiker traveled approximately 4 miles on the first day.

The equation for Sn, the sum of the first n terms of the sequence, is:

Sn = x(1 - r^n)/(1 - r)

where x is the first term, r is the common ratio (in this case, 1.1), and n is the number of terms.

Using the equation for Sn from Part B, we can find the total number of miles the hiker will travel in 14 days:

S14 = x(1 - 1.1^14)/(1 - 1.1)

S14 ≈ 167.63

Therefore, the hiker will travel approximately 167.63 miles in 14 days.

More on geometric series can be found here: https://brainly.com/question/4617980

#SPJ1

The most important of the Shinto gods is the sun goddess who gave light to the world, named ______.

Amaterasu
Susanoo
Tsukyomi
Izanagi

Answers

Answer: Amaterasu

Step-by-step explanation: The sun goddess Amaterasu is considered the most important of the Shinto gods because she is believed to be the ancestor of the Japanese imperial family, and therefore the protector of the Japanese people. She is also associated with agriculture, which was a vital part of Japanese society.

how to solve 3(x+6) = x + 8 + x

Answers

the solution to the equation  is x = -10.use the distributive property of multiplication over addition to simplify the left-hand side of the equation

what is distributive property ?

The distributive property is a mathematical property that is used to simplify expressions that involve multiplication and addition or subtraction. It states that when you multiply a number (or variable) by a sum or difference,

In the given question,

To solve the equation 3(x+6) = x + 8 + x, you can use the distributive property of multiplication over addition to simplify the left-hand side of the equation, and then combine like terms on both sides of the equation.

Here are the steps:

Distribute the 3 on the left-hand side of the equation:

3(x+6) = 3x + 18

Combine the two x terms on the right-hand side of the equation:

3x + 18 = 2x + 8

Subtract 2x from both sides of the equation:

3x - 2x + 18 = 2x - 2x + 8

Simplifying this expression gives:

x + 18 = 8

Finally, subtract 18 from both sides of the equation:

x + 18 - 18 = 8 - 18

Simplifying this expression gives:

x = -10

Therefore, the solution to the equation 3(x+6) = x + 8 + x is x = -10.

To know more about distributive property , visit:

https://brainly.com/question/5637942

#SPJ1

Determine whether the following statements are TRUE or FALSE (do not write down the statements
just state TRUE or FALSE). [7 marks]
a. () ≥ 1 for any event .
b. () = 1 where is the Sample space.
c. If {} is any finite or infinite sequence of disjoint events, then (⋃

=1 ) = ∑ ()
=1 .
d. If ⊆ where and are two events in a sample space, then () ≤ ().
e. If and are two events in a sample space, then ( ∪ ) = () − () + ( ∩ ).
f. If and are two independent events in a sample space, then ( ⁄ ) = (∩)
() .
g. Mutually exclusive events are not independent

Answers

a. TRUE, b. TRUE, c. TRUE, d. TRUE, e. TRUE, f. FALSE, g. TRUE

How to determine whether the following statements are TRUE or FALSE

a. TRUE: The probability of an event can never be negative, and can at most be equal to 1, which represents certainty.

b. TRUE: The sample space is the set of all possible outcomes of an experiment, and the probability of the sample space is always equal to 1, since one of the outcomes must occur.

c. TRUE: If the events in a sequence are disjoint, then they have no outcomes in common, so the probability of the union of the events is the sum of the probabilities of the individual events.

d. TRUE: If one event is a subset of another event, then the probability of the subset is less than or equal to the probability of the superset. This follows from the fact that the subset contains fewer outcomes than the superset.

e. TRUE: The probability of the union of two events is the probability of the first event plus the probability of the second event, minus the probability of the intersection of the events, which is the probability of both events occurring together. This is known as the inclusion-exclusion principle.

f. FALSE: The formula (P(A ∩ B) = P(A)P(B)) only applies to independent events, but not all independent events are mutually exclusive. For example, if A is the event of rolling a 4 on a die, and B is the event of rolling an even number, then A and B are independent, but not mutually exclusive.

g. TRUE: If two events are mutually exclusive, then they have no outcomes in common, so the occurrence of one event tells us that the other event cannot occur. This dependence means that the events are not independent.

Learn more about set theory at https://brainly.com/question/28916593

#SPJ1

A solid glass cylinder is 31 centimeters high and has a diameter of 3 centimeters. What is the mass of the cylinder if the density of glass is 1.6 grams per cubic centimeter?

Answers

Answer:

the mass of the solid glass cylinder is approximately 350.67 grams.

Step-by-step explanation:

The diameter of the cylinder is 3 centimeters, which means that its radius is 1.5 centimeters (half the diameter). The area of the base of the cylinder is the area of a circle, calculated as pi (π) multiplied by the radius squared:

Base area = π x radius^2

Base area = π x (1.5 cm)^2

Base area = 7.07 cm^2 (approximately)

The volume of the cylinder is calculated by multiplying the area of the base by the height:

Cylinder Volume = Base Area x Height

Cylinder volume = 7.07 cm^2 x 31 cm

Cylinder volume = 219.17 cm^3 (approximately)

Now that we know the volume of the cylinder, we can calculate its mass using the density of the glass:

Mass = Density x Volume

Mass = 1.6 g/cm^3 x 219.17 cm^3

Mass = 350.67 grams (approximately)

Help please i need this asap!! I'll give 100 points

Answers

The range is expressed in interval notation as (-1, ∞)

How to find the function (f+g)(x)?

To find the linear function f(x), let us use the table given.

A linear function with the following equation that passes through the points (a, g(a)) and (b, g(b)):

[tex]g(x) - g(a) = \frac{g(b)-g(a)}{b-a} (x-a)[/tex]

Because the g(x) line crosses through points (-6, 14) and (-3, 8), we have:

a = -6, g(a) = 16, b = -3 and, g(b) = 10

Therefore g(x)

[tex]g(x) - (16) = \frac{10-16}{-3-(-6)} (x--(6))\\g(x) - 16 = \frac{10-16}{-3+6} (x+6)\\g(x) - 16 = \frac{-6}{3} (x+6)\\g(x) - 16 = -2(x +6)\\g(x) = -2x -12+16\\g(x) = -2x+4[/tex]

now find the (f+g)(x).

[tex](f+g)(x) = f(x) + g(x) = x^{2} + 2x -5 -2x + 4\\(f+g)(x) = f(x) + g(x) = x^{2} - 1\\[/tex]

(f+g)(x) = (x-1)(x+1), therefore we get the values x = 1 and x = -1

The parabola's vertice has x-coordinate 0 (the midway between the roots). At x = 0, we get:

[tex](f +g)(x) = 0^{2} - 1 = -1[/tex]

Furthermore, because the coefficient of [tex]x^{2}[/tex] is 1, which is positive, this function indicates a parabola that has been opened upwards.

As a result, the function's minimal value is y = -1. As a result, the function's range includes all real numbers equal to or greater than -1.

The range is expressed in interval notation as (-1, ∞)

Learn more about function here:

https://brainly.com/question/12431044

#SPJ1

The table shows the age distribution of members of a gym. A member of gym is chosen at random. What is the probability that the person is: a) 21 or more b) 55 or less c) not in the 21 to 35 age group

Answers

The probability that the selected member is 21 or more is 88%.

The probability that the selected member is 55 or less is 86%.

The probability that the selected member is not in the 21 to 35 age group is 58%.

Finding probabilities:

The basic probability formula of dividing the number of favorable outcomes by the total number of outcomes.

In this case, we are given the percentage of members in each age group, and we need to find the probability of selecting a member with a certain age range.

Here we have

The table shows the age distribution of members of a gym.  

Age -            Under 21      21 -35     36 - 55   Over 55

percentage       12                42            32          14  

a) To find the probability that the selected member is 21 or more,

Add the percentage of members who are 21-35, 36-55, and over 55 since all of these age groups are 21 or more.

Probability (21 or more)

= Percentage (21-35) + Percentage (36-55) + Percentage (Over 55)

= 42% + 32% + 14%

= 88%

b) To find the probability that the selected member is 55 or less,  

Add the percentage of members who are under 21, 21-35, and 36-55 since all of these age groups are 55 or less.

Probability (55 or less)

= Percentage (Under 21) + Percentage (21-35) + Percentage (36-55)

= 12% + 42% + 32%

= 86%

c) To find the probability that the selected member is not in the 21 to 35 age group,

Add the percentage of members who are under 21, 36-55, and over 55 since all of these age groups are not in the 21 to 35 age group.

Probability (not in the 21 to 35 age group)

= Percentage (Under 21) + Percentage (36-55) + Percentage (Over 55)

= 12% + 32% + 14%

= 58%

Therefore,

The probability that the selected member is 21 or more is 88%.

The probability that the selected member is 55 or less is 86%.

The probability that the selected member is not in the 21 to 35 age group is 58%.

Learn more about Probability at

https://brainly.in/question/34187875

#SPJ9

A pianist plans to play 3 pieces at a recital from her repertoire of 25 pieces, and is carefully considering which song to play first, second, etc. to create a good flow. How many different recital programs are possible?

Answers

There are 13,800 different recital programs possible.

What is permutation?

In mathematics, a permutation is an arrangement of objects in a specific order. In other words, a permutation is a way of selecting a certain number of objects from a larger set and arranging them in a particular order.

The pianist has 25 choices for the first piece, then 24 choices for the second piece (since one piece has already been played), and 23 choices for the third piece (since two pieces have already been played). Therefore, the number of different recital programs possible is:

25 x 24 x 23 = 13,800

There are 13,800 different recital programs possible.

To learn more about permutation visit the link:

https://brainly.com/question/12468032

#SPJ1

Determine the effective tax rate for a taxable income of $115,500. Round theginal answer to the nearest hundredth
A) 18.71%
B) 17.20%
C) 24.10%
D) 24.75%

Answers

The effective tax rate for a taxable income of $115,500 is A) 18.71%

How to calculate the tax

The introductory $10,275 is subjected to a 10% tax burden, with the converted dollar amount representing $1,027.50 in taxes. The additional taxable sum of $30,900 ($41,175 - $10,275) is accessed at a 12% charge and aggregates to $3,708 worth of duties. An extra levy of 22% is imposed on the total $47,900 that lies between the two stipulated ranges ($89,075 - $41,175). The final evaluation stands at 24%, which provides an identical tax rate for the remaining $25,350 ($115,500 - $89,075). ).

Effective Tax Rate = (Total Tax Paid / Taxable Income) x 100%; which further articulates to ($21,357.50 / $115,500) x 100%,

= 18%

Learn more about tax on

https://brainly.com/question/25783927

#SPJ1

Find the area of the trapezoid 11 yd 11 yd 7 yd​

Answers

Answer:

Step-by-step explanation:

A=1/2(b1+b2)h

=1/2 (11yd+11yd)(7yd)

=1/2(22yd)(7yd)

=(11yd)(7yd)

=77yd

Given this equation what is the value of y at the indicated point?

Answers

Using curves, we can find that the value of y at the indicated point on the curve is √3.

What is the definition of curves?

A smooth-drawn figure or line with a bend or turns is referred to as a curve. A circle is an illustration of a curved shape. Geometry is a subfield of mathematics that examines the dimensions, characteristics, and shapes of figures.

Here in the question,

Given equation:

x = y² - 2

As a point (1, y) is on the curve, we can put the value of x coordinate in the equation:

1 = y² - 2.

Adding 2 on both sides:

1 + 2 = y² - 2 + 2

⇒ 3 = y²

⇒ y = √3.

Therefore, the value of y at the indicated point on the curve is √3.

To know more about curves, visit:

https://brainly.com/question/6436086

#SPJ1

In​ 2012, the population of a city was 5.51 million. The exponential growth rate was 3.82​% per year.
​a) Find the exponential growth function.
​b) Estimate the population of the city in 2018.
​c) When will the population of the city be 10 ​million?
​d) Find the doubling time.

helppppppp

Answers

Answer:

a) To find the exponential growth function, we can use the formula:

P(t) = P0 * e^(rt)

Where:

P(t) = the population at time t

P0 = the initial population (in this case, 5.51 million)

e = the mathematical constant e (approximately 2.71828)

r = the annual growth rate (in decimal form)

t = the number of years

Substituting the given values, we have:

P(t) = 5.51 * e^(0.0382t)

b) To estimate the population of the city in 2018, we can substitute t = 6 (since 2018 is 6 years after 2012) into the exponential growth function:

P(6) = 5.51 * e^(0.0382*6) ≈ 6.93 million

Therefore, the estimated population of the city in 2018 is approximately 6.93 million.

c) To find when the population of the city will be 10 million, we can set P(t) = 10 and solve for t:

10 = 5.51 * e^(0.0382t)

e^(0.0382t) = 10/5.51

0.0382t = ln(10/5.51)

t ≈ 11.7 years

Therefore, the population of the city will be 10 million in approximately 11.7 years from 2012, or around the year 2023.

d) To find the doubling time, we can use the formula:

T = ln(2) / r

Where:

T = the doubling time

ln = the natural logarithm

2 = the factor by which the population grows (i.e., doubling)

r = the annual growth rate (in decimal form)

Substituting the given value of r, we have:

T = ln(2) / 0.0382 ≈ 18.1 years

Therefore, the doubling time for the population of the city is approximately 18.1 years.

a. (10 points) The number of brownies sold by a bakery on a random day is a random variable with a mean value of 38 and a standard deviation of 6. What is the probability that the total number of brownies sold for a random sample of 56 days is less than 2070? Explain. b. (10 Points) Let X₁, X2, X3, and X4 represent the weight of shipment packages at a certain shipment facility. Suppose they are independent normal random variables with means µ4 7.4 pounds and μ₁ = 3.6, μ₂ = 0.9, µ3 = 1.8, μ4 µ2 variances o² = 0 = 0 = 0 = 1. Find P (2X₁ + 1X₂ + 3X3 + 1X4 ≤ 17.6). -​

Answers

a. The probability that the total number of brownies sold for a random sample of 56 days is less than 2070 is approximately 0.0107.

b. The probability that 2X₁ + X₂ + 3X₃ + X₄ is less than or equal to 17.6

Explain probability

Probability is the measure of the likelihood or chance of an event occurring. It is expressed as a number between 0 and 1, where 0 means the event will not occur, and 1 means the event will definitely occur. Probability is a fundamental concept in mathematics and is used in many fields, including statistics, science, and finance.

According to the given information

a. The total number of brownies sold for a random sample of 56 days, Y, is a normal random variable with mean µ_Y = 56µ_X = 5638 = 2128 and standard deviation σ_Y = √(56)*σ_X = √(56)*6 = 25.21.

We want to find P(Y < 2070). Standardizing Y, we get:

Z = (Y - µ_Y) / σ_Y = (2070 - 2128) / 25.21 = -2.31

Using a standard normal distribution table or calculator, we can find that P(Z < -2.31) is approximately 0.0107. Therefore, the probability that the total number of brownies sold for a random sample of 56 days is less than 2070 is approximately 0.0107.

b. 2X₁ + X₂ + 3X₃ + X₄ is also a normal random variable with mean 2µ₁ + µ₂ + 3µ₃ + µ₄ = 2(3.6) + 0.9 + 3(1.8) + 7.4 = 18.5 and variance 4o² + o² + 9o² + o² = 14o².

We want to find P(2X₁ + X₂ + 3X₃ + X₄ ≤ 17.6). Standardizing the variable, we get:

Z = (17.6 - 18.5) / √(14o²) = -0.5735 / √(o²)

To find P(Z ≤ -0.5735 / √(o²)), we need to know the value of o². If o² = 1, then P(Z ≤ -0.5735) = 0.2831. If o² is a different value, we would need to adjust accordingly.

Therefore, the probability that 2X₁ + X₂ + 3X₃ + X₄ is less than or equal to 17.6

To know more about probability visit

brainly.com/question/11234923

#SPJ1

Naya has a pitcher that contains 3 cups of salted lassi, a yogurt drink with sait and sites. She pours 6 fluid ounces of lassi into each glass. If she uses all of the lassi, how many glasses does Naya use?
A. 2
B. 4
C. 16
D. 18

Answers

After 6 fluid ounces , Naya uses 4 glasses as a result.

Define ounces?

A unit of weight is an ounce. There are various kinds of ounces, including avoirdupois, troy, and fluid ounces. One sixteenth of a pound is equivalent to one avoirdupois ounce .  A troy ounce, often known as an apothecaries' measure, is equivalent to 480 grains or one-twelfth of a pound. A volume unit is a fluid ounce. 1/8 of a cup, 2 tablespoons, or 6 teaspoons make to one fluid ounce

In Naya's pitcher, there are three glasses of salted lassi.

She fills each glass with six fluid ounces of lassi.

By translating cups to fluid ounces and dividing the entire amount of lassi by the amount put into each glass, we can determine how many glasses Naya uses if she consumes all of the lassi.

8 fluid ounces make constitute a cup.

Consequently, 3 cups equal 24 fluid ounces (3 x 8).

24 divided by 6 results in:

4 glasses are equal to 24/6.

Naya uses four glasses as a result.

To know more about ounces visit:

brainly.com/question/29374025

#SPJ1

ΔABC with vertices A(-3, 0), B(-2, 3), C(-1, 1) is rotated 180° clockwise about the origin. It is then reflected across the line y = -x. What are the coordinates of the vertices of the image?

A.
A'(0, 3), B'(2, 3), C'(1, 1)

B.
A'(0, -3), B'(3, -2), C'(1, -1)

C.
A'(-3, 0), B'(-3, 2), C'(-1, 1)

D.
A'(0, -3), B'(-2, -3), C'(-1, -1)

Answers

The coordinates of the vertices of the image is A'(0, -3), B'(3, -2), C'(1, -1). The correct option is B.

What are coordinates?

Triangle ABC is rotated 180 degrees clockwise about the origin and then reflected across the line y=-x.

We are to find the coordinates of the vertices of the image.

We know that

if a point (x, y) is rotated 180 degrees clockwise, then its co-ordinate changes as follows :

(a, b)   ⇒    (-a, -b).

So, after getting rotated 180 degrees clockwise, the coordinates of the vertices of triangle ABC becomes

A(-3, 0)   ⇒  (3, 0)

B(-2, 3)   ⇒   (2, -3)

C(-1, 1)    ⇒   (1, -1).

Therefore, the correct option is B. A'(0, -3), B'(3, -2), C'(1, -1).

Learn more about coordinates, here:

https://brainly.com/question/5961156

#SPJ1

At Amy's Pizza Palace, the cost of a pizza depends on the number of toppings. The graph shows this relationship.
247
21
Cost of Pizza
(in $)
18-
15-
12-
9
6-
3-
1
2
6
3 4
Number of Toppings
7
TURUDANY
According to this graph, what is the meaning of the v-intercept?

Answers

The cost of pizza with no toppings is $6. So correct option is C.

Describe Graph?

A graph is a visual representation of data or information, typically shown on a coordinate plane or a network of nodes and edges. Graphs are used to help people understand complex information by presenting it in a clear and concise way.

There are many different types of graphs, including bar graphs, line graphs, scatter plots, pie charts, and network graphs. Each type of graph is best suited for representing different types of data. For example, bar graphs are used to represent discrete data or data that can be divided into categories, while line graphs are used to represent continuous data or data that changes over time. Scatter plots are used to represent the relationship between two variables, while pie charts are used to represent percentages or proportions. Network graphs are used to represent complex systems of relationships between objects or people.

Graphs are commonly used in fields such as business, economics, science, and engineering to help people understand trends, patterns, and relationships in data. They can also be used to make predictions, identify outliers or anomalies, and communicate results to others. In addition, graphs can be customized to suit the needs of different audiences, such as by adding labels, titles, and other visual elements to make the data more understandable.

Because when x is 0 ,y is 6. So, the y intercept is the cost of a pizza with no toppings is $6.

To know more about intercept visit:

https://brainly.com/question/31350827

#SPJ1

The complete question is:

Find any solution(s) (refer to attachment) of and select the correct statement.

A. The equation has no solution.

B. The equation has two solutions.

C. The equation has one solution.

D. The equation has one solution and one extraneous solution.

Answers

B
x+2=0
x-1=0
x≠-1
x1=-2, x2=1

HELP RAAHHHH

1. During halftime of a football game, a sing shot launches T-shirts at the crowd
A T-shirt is launched from a height of 4 feet with an intal upward velocity of 72 feet per second
The T-shirt is caught 42 feet above the field
How long will take the T-shirt to reach its maximum height? What is the maximum height? What is the range of the function that models the height of the T-shirt over time?

2. During halftime of a football game, a sing shot launches T-shirts at the crowd
A T-shirt is launched from a height of 3 feet with an intal upward velocity of 80 feet per second
The T-shirt is caught 36 feet above the field
How long will take the T-shirt to reach its maximum height? What is the maximum height? What is the range of the function that models the height of the T-shirt over time?

Answers

Answer:

1. Using the kinematic equation h(t) = -16t^2 + v0t + h0, where h0 is the initial height, v0 is the initial velocity, and t is time, we have:

h(t) = -16t^2 + 72t + 4

To find the maximum height, we need to find the vertex of the parabolic function h(t). The t-coordinate of the vertex is given by t = -b/2a, where a = -16 and b = 72:

t = -b/2a = -72/(2(-16)) = 2.25 seconds

To find the maximum height, we substitute t = 2.25 seconds into the equation for h(t):

h(2.25) = -16(2.25)^2 + 72(2.25) + 4 = 82 feet

The range of the function h(t) is [4, 82], since the T-shirt starts at a height of 4 feet and reaches a maximum height of 82 feet before falling back to the ground.

2. Using the same kinematic equation as before, we have:

h(t) = -16t^2 + 80t + 3

To find the maximum height, we again need to find the vertex of the parabolic function h(t). The t-coordinate of the vertex is given by t = -b/2a, where a = -16 and b = 80:

t = -b/2a = -80/(2(-16)) = 2.5 seconds

To find the maximum height, we substitute t = 2.5 seconds into the equation for h(t):

h(2.5) = -16(2.5)^2 + 80(2.5) + 3 = 80 feet

The range of the function h(t) is [3, 80], since the T-shirt starts at a height of 3 feet and reaches a maximum height of 80 feet before falling back to the ground.

Step-by-step explanation:

Charlie invests $325 in an account that pays 8% simple interest for 15 years.

Use the simple interest formula, I = P ∙ r ∙ t, to answer the following questions.



How much interest will Charlie’s initial investment earn over the 15-year period?


How much money does Charlie have after the 15 years?

Answers

Answer:          To answer these questions using the simple interest formula, we need to know the values of P (the principal or initial investment), r (the interest rate), and t (the time period in years).

In this case, P = $325, r = 0.08 (8% expressed as a decimal), and t = 15.

Using the simple interest formula, I = P ∙ r ∙ t, we can calculate:

I = $325 ∙ 0.08 ∙ 15 = $390

Therefore, Charlie's initial investment will earn $390 in interest over the 15-year period.

To calculate how much money Charlie will have after the 15 years, we need to add the interest earned to the initial investment.

The total amount of money that Charlie will have after the 15 years is:

Total amount = Initial investment + Interest earned

Total amount = $325 + $390

Total amount = $715

Therefore, Charlie will have $715 after 15 years.

Step-by-step explanation:

Other Questions
Why was the Warren Commission created in November 1963?to prevent presidential assassinationsto explore Thurgood Marshalls appointmentto investigate Kennedys assassinationto train future Secret Service agents 6) Baldwin Corp. just paid a dividend of $2.00. Over the next two years, this dividend is expected to grow by 20% per year. After two years, dividend growth is expected to level off at 10%. If the required rate of return on Baldwin stock is 12%, what should be the price of Baldwin stock today? a generalized (tonic-clonic) seizure is characterized by: emt If a vehicle starts to skid on water (hydroplane), the driver should ease off the accelerator, brake gently and gently steer back onto the pavement. (true or false) 2. Define merit-based aid. For what achievement or ability are most merit-based scholarshipsawarded? (2 points) Cardiac output may decrease if the heart beats too rapidly because:A.a rapid heart beat causes a decrease in the strength of cardiac contractions.B.the volume of blood that returns to the heart is not sufficient with fast heart rates.C.as the heart rate increases, more blood is pumped from the ventricles than the atria.D.there is not enough time in between contractions for the heart to refill completely. Aaron received a 30 year loan of $315,000 to purchase a house. The interest rate on the loan was 4.10% compounded semi-annually.a. What is the size of the monthly loan payment?Round to the nearest centb. What is the balance of the loan at the end of year 3?Round to the nearest centc. By how much will the amortization period shorten if Aaron makes an extra payment of $30,000 at the end of year 3? Pls help me fill out his chart due tomorrow at 3:30pm in 2003, the u.s. department of justice banned its practice of racial profiling in all federal law enforcement agencies, except in cases involving ________. which could constitute a second class of stock? group of answer choices treasury stock phantom stock. unexercised stock options. warrants. none of the above. when then number of needed items are computed based on the number of higher-level items produced, one is operating in a(n) as the afterload is increased, how did the latency change which of the following costs is deductible as an itemized medical expense?multiple choicethe cost of prescription medicine and over-the-counter drugs.medical expenses incurred to prevent disease.the cost of elective cosmetic surgery.medical expenses reimbursed by health insurance.none of the costs are deductible. sponsorship agreements within the sport industry commonly refer to the team, event, or sport organization as the . a. intermediary b. brand c. property d. agency a client is receiving an iv solution of 2 grams of medication diluted in 100 ml of normal saline over a one hour time period. how many mg of medication is the client receiving per minute? (enter numeric value only. if rounding is required, round to the nearest whole number.) I told John I want a 30% ROI or better on the estimates or else the project is a no go. "Prove it to me in a business case John. Then well run with your idea." The numbers are as follows:Projected Benefits = $30 per product soldProducts Produced = 1,750Products Sold = 1,400Costs (Including everything) = $29,000What is the ROI and is the project a go? Show all work. a client is to be started on an antibiotic. which is most important to take into consideration before beginning the antibiotic regimen? this exquisite, light-as-air track from magical mystery tour - featuring flutes, bass harmonicas, and a recorder played by paul mccartney Why are a growing number of constitutional scholars, lawyers and judges questioning its wisdom? Exclusionary rule: science: Dr. Alex Young would like to know about your progress in solving the mystery about the rough-skinned newts. In the space below, answer the Chapter 1 Question: What caused this newt population to become more poisonous?Use what you have learned so far to consider these claims:Claim 1: Individual newts became more poisonous because they wanted to.Claim 2: The newt population became more poisonous because of something in the environment.Revised Claim 2: The newt population became more poisonous because the snakes in this environment caused poison to be an adaptive traitChoose one or more claims to support with evidence and reasoning. You may also want to explain why one or more of the claims is definitely not correct.Be sure to use some of the vocabulary words you have learned so far:adaptive traitdistributionenvironmentnon-adaptive traitpopulationtraitvariation